Calendrier de l’Avent

2456789

Réponses

  • Merci Calli d'avoir détecter la faute d'inattention. 
    Le 😄 Farceur


  • @Calli Merci. C'est ok pour dimanche avant 11h.
  • Jour 3

    Soit $x \in{\mathbb{R}}$. Calculer $\displaystyle \tan\left(\sum_{k = 0}^{+\infty} \arctan\left(\frac{4 (-1)^k}{2 k + 1} x\right)\right)$.
  • Je crois que c'est quelque chose du style $1/\text{sh}(2\pi x)$ mais j'ai peur de m'être embrouillé à la fin des calculs.

    Mais je ne vois pas l'intérêt du 4 dans l'énoncé. 
  • etanche
    Modifié (December 2022)
    @ Calli la formule pour x=0 donné 0, ton résultat n’est pas défini pour x=0. Y a peut-être un oubli.
  • bd2017
    Modifié (December 2022)
    $\tanh(\pi x)$
    Je trouve ceci.
     
  • Bibix
    Modifié (December 2022)
    Le $4$ permet de faire disparaitre la fraction dans le résultat trouvé par bd2017. Moi, je préfère le noter :
    $\displaystyle \tan\left(\sum_{k = 0}^{+\infty} \arctan\left(\frac{4 (-1)^k}{2 k + 1} x\right)\right) = \tanh\left(\sum_{k = 0}^{+\infty} \frac{4 (-1)^k}{2 k + 1} x\right)$

    Je trouve le résultat plus intéressant sous cette forme...

  • etanche
    Modifié (December 2022)
    Comment faire pour poster une réponse cachée en mode "Révéler" ? Merci.
  • Bouton ¶, dernière option. 
  • Bonjour, suite à une demande d'etanche, une solution complète devra être publié le lendemain du jour par l'auteur de l'exo :) Si l'auteur a le temps de le faire bien évidemment 
    Je suis donc je pense 
  • Boécien
    Modifié (December 2022)
    Cher champion @Quentino37 il faudrait que tu commences à mettre des liens de ton premier message vers les énoncés et solutions des problèmes posés, un peu comme dans le fil de ton maitre @gebrane sur Cesàro. Sinon ça va vite devenir difficile de naviguer dans ce fil.
  • gebrane
    Modifié (December 2022)
    @Boécien Bonne idée
    @Bibix sais-tu que tu as tué l'exercice en donnant ta réponse ? Calli a donné un résultat faux (une erreur dans son calcul) et et bd a donné juste une réponse mais toi tu as révélé le point essentiel .
     Peut-on se fixer une heure pour envoyer une réponse car certains lisent les révélés et les commentent ici ce qui peut troubler  notre concentration.
    Le 😄 Farceur


  • Est-il prévu un pdf avec les tous les énoncés et solutions ? Merci 
  • C’est assez culotté toutes ces formes d’exigence… et pourquoi pas le faire, toi, etanche, ce pdf  ?



  • @Dom mais voyons, c'est étanche qui se porte volontaire pour ce pdf et même il peut créer un fil en parallèle pour mettre les questions et réponses en ordre 
    Le 😄 Farceur


  • Calli
    Modifié (December 2022)
    En fait, j'avais trouvé (avec une mise à l'inverse en trop) la valeur de $\displaystyle \tan\left(2\sum_{k = 0}^{+\infty} \arctan\left(\frac{4 (-1)^k}{2 k + 1} x\right)\right)$ avec un 2 en plus dans la tangente, je n'avais pas fait gaffe. Mais j'ai finalement trouvé comme bd2017 pour l'expression demandée. Par contre, ma preuve est compliquée je trouve ; j'espère qu'il y a plus simple.
    Ok @Bibix je comprends mieux.
  • @Quentino37 : Quand une réponse complète a été donnée (comme c'est le cas hier), il est inutile que l'auteur de l'exercice reposte une correction.

    Merci @etanche de t'être porté volontaire pour faire ce pdf. On compte sur toi. :mrgreen:
  • Quentino37
    Modifié (December 2022)
    Je suis donc je pense 
  • Je ne sais pas mais @gebrane doit savoir!
  • Ce n'est pas pratique sur un téléphone. 
    Le 😄 Farceur


  • Je n'aurais pas le temps de jouer malheureusement mais je veux bien être volontaire pour le pdf s'il n'y a pas trop d'exigence de rapidité.
  • etanche
    Modifié (December 2022)
    Une remarque les $f(k)=\sum_{n=1}^{+\infty} \frac{(-1)^{nk}}{(2n+1)^k}$ se calculent suivant la parité de $k$ avec les nombres de Bernoulli $B_k$, eulerien $E_k$ et $\pi^k$.
  • Calli
    Modifié (December 2022)
    Voici ma solution. Ça fera peut-être rire ceux qui en auraient une plus courte de me voir ramer ainsi, mais ce n'est pas grave !
    Soit $\displaystyle f(x) :=  \sum_{k=0}^\infty  (-1)^{k} \arctan\!\left( \frac{x}{2k+1} \right)$. Comme je n'ai aucune idée a priori du résultat et pas d'inspiration particulière, je vais calculer la dérivée seconde de $f$ puis primitiver deux fois. Tout d'abord \[\frac{\mathrm{d}f}{\mathrm{d}x} = \sum_{k=0}^\infty  (-1)^{k} \frac{\mathrm{d}}{\mathrm{d}x}  \arctan\!\left( \frac{x}{2k+1} \right) = \sum_{k=0}^\infty  (-1)^{k} \frac{2k+1}{(2k+1)^2 +x^2 }.\] Pour le justifier, on peut regrouper les termes deux par deux et vérifier que $\frac{2k-1}{(2k-1)^2 +x^2 } - \frac{2k+1}{(2k+1)^2 +x^2 } =O( \frac{1}{k^2 } )$, avec un $O$  localement uniforme en $x$. Ensuite, en posant $y=\mathrm{i}x$ on a \[\frac{\mathrm{d}f}{\mathrm{d}x} = \sum_{k=0}^\infty  (-1)^{k} \frac{2k+1}{(2k+1)^2 -y^2 } = \frac{1}{2} \sum_{k=0}^\infty  (-1)^{k} \left( \frac{1}{y+2k+1} - \frac{1}{y-(2k+1)} \right).\] Puis on redérive : \[\frac{\mathrm{d}}{\mathrm{d}y} \frac{\mathrm{d}f}{\mathrm{d}x} = \frac{1}{2} \sum _{k\in \mathbb{Z}} \frac{(-1)^{k} }{(y-(2k+1))^2 } \overset{(*)}= \frac{\pi ^2 }{8} \left( \frac{\sin }{\cos ^2 } \right)\!\left( \frac{\pi }{2} y\right).\] Pour justifier l'égalité $(*)$, on remarque que ses deux membres sont méromorphes sur $\mathbb{C}$, que leurs pôles sont les entiers relatifs impairs, qu'au voisinage de tout pôle $p$ ils valent $ \frac{1}{2(y-p)^2 } +O(1)$, qu'ils sont $2\pi $-périodiques et qu'ils tendent vers 0 quand $|\Im(y)| \rightarrow \infty $. Ainsi, leur différence est holomorphe sur $\mathbb{C}$ et bornée, donc elle est constante (nulle ici). On a justifié $(*)$.
    Ensuite, on primitive : \[\displaystyle \frac{\mathrm{d}f}{\mathrm{d}x} = \frac{\pi }{4} \cos \!\left( \frac{\pi }{2} y\right)^{-1} = \frac{\pi }{4} \cos \!\left( \frac{\pi }{2} \mathrm{i}x\right)^{-1}\] avec une constante de primitivation nulle en évaluant en 0 ou en prenant la limite $x\rightarrow \infty $. Puis, sachant que la primitive de $ \frac{1}{\cos t}$ est $\log  \tan ( \frac{t}{2} + \frac{\pi }{4} )$, \begin{eqnarray*} f(x) &=& - \frac{\mathrm{i}}{2} \log  \tan \!\left( \frac{\pi }{4} \mathrm{i}x + \frac{\pi }{4} \right) +c \\ &=& - \frac{\mathrm{i}}{2} \log \!\left( \frac{1+\mathrm{i}e^{\pi x/2} }{1-\mathrm{i}e^{\pi x/2} } \right) +c \\ &=& \arg(1+\mathrm{i}e^{\pi x/2} ) +c\\ && \text{ car } 1+\mathrm{i}e^{\pi x/2} \text{ et } 1-\mathrm{i}e^{\pi x/2} \text{ sont conjugués}\\ &=& \arctan(e^{\pi x/2} ) +c \end{eqnarray*} En évaluant en $x=0$ on voit que $c=- \frac{\pi }{4}$. Enfin le résultat demandé est $\tan  f(4x) = \operatorname{th}(\pi x)$ par la formule de $\tan (a-b)$.
  • etanche
    Modifié (December 2022)
    @ rémi il n’y aucune exigence de rapidité, merci pour le pdf, que beaucoup de lecteurs apprécieront certainement. 

  • Cidrolin
    Modifié (December 2022)
    Je suis d'accord pour poser un problème le jour correspondant au plus petit entier non nul qui n'est pas un $v_n$, avec
    $v_n=\dfrac{1}{100}\big \lfloor\dfrac{5000n-1}{4998}\big \rfloor$
  • @Cidrolin je crois que je vais prendre mon tableur.
  • Bonjour,

    Un petit programme en Python montre que c'est Noël  :).

    Cordialement,
    Rescassol

  • Jour 4

    Soient $p$ et $q$ deux nombres premiers $\geq 3$.
    Trouver une condition nécessaire et suffisante pour que le morphisme $\displaystyle f_p : \mathbb F_q^* \rightarrow \mathbb F_q^*, x \mapsto x^p$ soit un automorphisme.
    Cas particulier : pour $q \equiv -1 \pmod p$, résoudre l'équation $x^p=a$ dans $\mathbb F_q^*$.
  • @Calli jolie solution. Je comprends mieux pourquoi Bibix a laissé trainer un x! Et le coup de la fonction complexe constante, fallait y penser. Il y a une autre solution foncièrement différente?
  • Le jour 4 est le jour de  @ev :|
    Le 😄 Farceur


  • Tu as un décalage horaire @Julia Paule ? :mrgreen: Chez moi on est encore le 3 décembre.
  • Merci @Boécien. Je n'ai pas inventé l'idée, j'ai repris la technique de preuve de $\displaystyle\sum_{n\in\Bbb Z}\frac1{(z-n)^2} =\frac{\pi^2}{\sin^2(\pi z)}$ que je connais.

    @Bibix est-ce que tu pourrais donner ta solution s'il te plaît ? J'aimerais savoir s'il y a une astuce qui permet d'éviter ce que j'ai fait.
  • Quentino37
    Modifié (December 2022)
    Ok :) ton problème devra être l’un des plus beau de tous ;)
    PS. Quel est la probabilité que deux personnes aient choisi le même problème ? 
    Je suis donc je pense 
  • On peut donner une solution à l'exo de @Julia Paule ?
  • Calli
    Modifié (December 2022)
    Je pense que oui @gai requin, mais cache ta réponse dans un environnement "révéler" (accessible avec le bouton ¶) pour laisser réfléchir ceux qui le veulent.  :)
  • gebrane
    Modifié (December 2022)
    Jour4
    $f_p$ est bijective ssi l'equation $x^p=1$ admet une unique solution dans $\mathbb F_q^*$ ssi $pgcd (p,q-1)=1$.
    Le 😄 Farceur


  • Il faudrait un système de plis cachetés comme à l'Académie des sciences. Du moins mieux cachetés que par le bouton "révéler".
  • gai requin
    Modifié (December 2022)
    Jour 4
    Comme $\mathbb F_q^*$ est cyclique, $f_p$ n'est pas un automorphisme ssi $f_p$ n'est pas injectif ssi $\mathbb F_q$ admet un élément d'ordre $p$ ssi $p$ divise $q-1$.
    Si $q\equiv -1\bmod p$, $f_p$ est donc un automorphisme et l'unique antécédent de $a\in\mathbb F_q^*$ est $a^{\tfrac{q-1}2}\times a^{\tfrac{q+1}{2p}}$.
    Sinon, j'ai en boutique un bel exo d'arithmétique style olympique peut-être inédit...
  • Bibix
    Modifié (December 2022)
    Tu vas être assez déçu @Calli car je ne connais pas moi-même de démonstration élémentaire. On peut néanmoins se reposer un peu moins (en apparence du moins) sur l'analyse complexe au prix d'un peu plus de calculs :
    On pose $\displaystyle f : x \longmapsto \sum_{k = 0}^{+\infty} (-1)^k \arctan\left(\frac{x}{2 k + 1}\right) = \sum_{k = 0}^{+\infty} (-1)^k a_k(x)$.

    C'est une série alternée de fonctions, et telle que $\sum_{k = 0}^{+\infty} (-1)^k a_k'(x)$ est aussi une série alternée à partir d'un certain rang $N_x$ qui est croissant en fonction de $x^2$ donc la série dérivée de fonctions converge uniformément sur tout compact. Donc $f \in C^1(\mathbb{R})$ et on a :
    \begin{equation*}
        \forall x \in \mathbb{R}, f'(x) = \sum_{k = 0}^{+\infty} (-1)^k a_k'(x) = \sum_{k = 0}^{+\infty} \frac{(-1)^k (2 k + 1)}{(2 k + 1)^2 + x^2}
    \end{equation*}

    Soit $x \in \mathbb{R}$, on considère la série de Fourier de $g_x : t \longmapsto e^{x t}$ sur $[-\pi, \pi]$ ce qui donne (après un petit calcul)
    $$
        \forall t \in ]-\pi, \pi[, g_x(t) = \frac{\sinh(\pi x)}{\pi x} + \frac{\sinh(\pi x)}{\pi}\sum_{k \in \mathbb{Z}^*} \frac{(-1)^k}{x^2 + k^2} (x + i k) e^{i k t}.
    $$
    On obtient alors la formule valable pour tout $x \in \mathbb{R}^*$
    $$
        \frac{\pi}{2 \sinh(\pi x)} g_x(0) - \frac{1}{2 x}= \sum_{k = 1}^{+\infty} \frac{(-1)^k x}{x^2 + k^2} = \frac{\pi}{2 \sinh(\pi x)} - \frac{1}{2 x}, \quad (*)
    $$
    ainsi que
    $$
        \frac{\pi}{2 \sinh(\pi x)} g_x\left(\frac{\pi}{2}\right) - \frac{1}{2 x} = \sum_{k = 1}^{+\infty} \frac{(-1)^k x}{x^2 + (2 k)^2} + \sum_{k = 0}^{+\infty} \frac{(-1)^k (2 k + 1)}{x^2 + (2 k + 1)^2} = \frac{\pi e^{\dfrac{\pi x}{2}}}{2 \sinh(\pi x)} - \frac{1}{2 x}.
    $$ Or d'après $(*)$, on a
    $$
        \sum_{k = 1}^{+\infty} \frac{(-1)^k x}{x^2 + (2 k)^2} = \frac{1}{2} \sum_{k = 1}^{+\infty} \frac{(-1)^k \left(\frac{x}{2}\right)}{\left(\frac{x}{2}\right)^2 + k^2} = \frac{1}{2}\left(\frac{\pi}{2 \sinh\left(\frac{\pi}{2} x\right)} - \frac{1}{x}\right),
    $$
    d'où
    \begin{align*}
        \sum_{k = 0}^{+\infty} \frac{(-1)^k (2 k + 1)}{x^2 + (2 k + 1)^2} = \frac{\pi e^{\dfrac{\pi x}{2}}}{2 \sinh(\pi x)} - \frac{\pi}{4 \sinh\left(\frac{\pi}{2} x\right)} & = \frac{\pi}{4 \sinh\left(\frac{\pi x}{2}\right)} \left(\frac{e^{\dfrac{\pi x}{2}}}{\cosh \left(\frac{\pi x}{2}\right)} - 1\right) \\
        & = \frac{\pi}{4 \sinh\left(\frac{\pi x}{2}\right)} \tanh\left(\frac{\pi x}{2}\right) \\
        & = \frac{\pi}{4 \cosh\left(\frac{\pi x}{2}\right)} \\
        & = \frac{\pi}{2} \frac{e^{\frac{\pi x}{2}}}{e^{\pi x} - 1}.
    \end{align*}
    On a donc
    \begin{equation*}
        f(x) = \int_0^x f'(t) dt = \int_0^x \frac{\pi}{2} \frac{e^{\frac{\pi t}{2}}}{e^{\pi t} - 1} dt = \arctan\left(e^{\frac{\pi x}{2}}\right) - \frac{\pi}{4}.
    \end{equation*}
    D'où le résultat
    \begin{equation*}
        \tan f(4 x) = \tanh(\pi x).
    \end{equation*}

    J'ai toujours trouvé ce résultat intriguant... c'est pour cela que je préfère le noter dans sa version où on voit que les $\arctan$ disparaissent et $\tan$ devient $\tanh$... mais comme en général c'est faux, j'ai laissé tomber.

    Edit : j'ai corrigé l'erreur signalée par JLapin.
  • Julia Paule
    Modifié (December 2022)
    @gebrane Le jour 4 est bien mon jour, et il y a deux questions dans mon exo.
    Avec un peu d'avance @Calli ! Si mon exo est trop facile, tu peux en mettre un autre.
    Pour le jour 3, il doit quand même y avoir une solution plus simple. L'équivalent $\arctan (\dfrac{x}{2k+1}) \sim \dfrac{x}{2k+1}$ quand $k \rightarrow +\infty$, donne $\tan(\pi x)$ au lieu de $\tanh(\pi x)$. Amusant.
  • JLapin
    Modifié (December 2022)
    Jour 3
    Je dérive (sans la tangente), je reconnais une sorte de "somme eulérienne" qui se simplifie, j'intègre puis je passe à la tangente et je trouve $th(\pi x)$.
  • @Julia Paule : J'ai aussi résolu la deuxième question ;)
  • JLapin
    Modifié (December 2022)
    Merci pour les calculs détaillés ! Par contre, tu pourrais plutôt mentionner la convergence uniforme de la série des dérivées :)
  • En effet @gai requin C'est ça. Toujours ce problème des messages qui apparaissent 10mn après.
  • Julia Paule
    Modifié (December 2022)
    Pour le jour 3, la relation $\tan(ix)=i \tanh(x)$ doit expliquer le lien.
    @etanche, la solution du jour 4 dans math.stackechange parait compliquée (je n'ai pas regardé le détail), mais la question est plus générale. Merci pour ce lien, je regarderai demain.
  • gebrane
    Modifié (December 2022)
    Bon matin @Julia Paule6:39
    Tu n'as pas compris mon message quand j'avais dit que c'est le jour de ev. J'ai pensé  que c'est le jour où va briller ev, avec une élégante preuve, c'est sa spécialité. 
    Peut-on faire les prochains jours comme étanche ?  Au lieu de travailler avec nos mains, on travaille avec la souris !
    Le 😄 Farceur


  • Bonjour @gebrane,
    Tu me fais trop rire. Ah c'est la spécialité de ev, dans ce cas, il n'a eu besoin ni de la souris ni de ses mains, de tête sûrement.
    @gai requin, et ce joli exo d'arithmétique ?
  • @Bibix

    Dans ta démonstration on peut aller un peu plus vite en calculant la série de Fourier de $h_x : t \longmapsto \sinh(x t)$ sur $]-\pi,\pi[$ 

    (pour une fonction impaire il n'y a que les $b_n$ à calculer).

    On obtient : $\forall t \in ]-\pi, \pi[, \dfrac{\pi\sinh(x t)}{2\sinh(x\pi)} =\displaystyle\sum_{n=1}^{+\infty} \frac{(-1)^{n-1}n\sin(nt)}{x^2 + n^2}$ qui donne directement pour $t=\dfrac{\pi}2$ : $$ \sum_{k = 0}^{+\infty} \frac{(-1)^k (2 k + 1)}{x^2 + (2 k + 1)^2} =\dfrac{\pi}{4 \cosh\left(\dfrac{\pi x}{2}\right)}$$

  • @Julia Paule : Si j’ai bien compris les règles, je dois m’inscrire sur un créneau et surtout disposer d’une solution 😏
Connectez-vous ou Inscrivez-vous pour répondre.